PT 86 - Multiverse - Q20 Forum

Prepare for the LSAT or discuss it with others in this forum.
Post Reply
LSATMoron

New
Posts: 4
Joined: Sun Feb 24, 2019 10:20 am

PT 86 - Multiverse - Q20

Post by LSATMoron » Wed Oct 09, 2019 8:47 pm

Can anyone please give me specific advice as to where im going wrong in understanding this passage because I keep landing on answer D for the main point question instead of answer A.

P1 - Physicists posit that the starting point for our universe was one of low entropy. Then we experienced the Big Bang.

P2 - C&C argue that the Big Bang was not a unique event, events like this happen from time to time.

P3- Based on the second law of thermodynamics, things naturally move from low entropy (less chaos) to high entropy (more chaos).

P4 - (Now the author ties all the three previous paragraphs). Sure we can agree with the aforementioned physicists on the Big Bang but it seems unlikely that the starting point for our universe would be one of low entropy. Why? because the multiverse came before our universe and since things only go from low entropy to high entropy, one would expect the starting point for our universe (in contrast to that of the multiverse) to be of high entropy. C & C's innovation is to argue that the initial starting point for our universe was cold and empty not hot and dense like the physicists argued.

But there is a problem with C & C's theory, can cold empty space lead to massive inflation?

P5- Recent research reveals yes it can due to energy fluctuations on the subatomic scale! C & C theorize our universe, and others, is the result of such a fluctuation.

P6 - Author Agrees
When I look at P5, it seems the primary purpose of the paragraph is to resolve one problem that is raised by C & C's theory in the previous paragraph. i.e to explain how cold empty space can lead to massive inflation. In other words, paragraph five exists to support paragraph four. So naturally I would think the main point is in P4 and not P5.

Any help will be greatly appreciated.

Thank you!

Blueprint LSAT
Posts: 77
Joined: Thu Jan 31, 2019 5:43 pm

Re: PT 86 - Multiverse - Q20

Post by Blueprint LSAT » Thu Oct 10, 2019 4:44 pm

I definitely see your logic here. P5 does sound a lot like they are just clearing up a potential problem with the theory. I think the best way to get where we need to go is to look at which of the answer choices is the best summary of the point C&C are trying to make.

Here are all the claims made by C&C:
P2: The Big Bang was not a unique event; events like it happen periodically over an incredibly vast time
scale.
P4: The most common initial condition is actually likely to resemble cold, empty space.
P5: Our universe and others are fluctuations in a high entropy multiverse.

If you think of the statement in P5 as just summarizing how they are solving the problem presented in P4 then doesn't sound like a good candidate for the main point.

However, if you think of it as an updated paraphrasing of the overall claim after accounting for that problem and solution it makes a little more sense I think.

We have two seemingly contradictory hypotheses:
The starting conditions should be low-entropy
The Big Bang was not a unique event

Some evidence is added to reconcile these two expectations and the explanation we are left with is that our universe and others are fluctuations in a high entropy multiverse. That is the explanation the author goes on to agree with, which makes A the best candidate among our answer choices for our main point.

I don't love it but it checks.

The problem with D is that even if A was support for it it would then still be support for something else (probably the original claim in P2). C&C's overall claim isn't that high entropy is the likeliest starting condition. That is a complication they became interested in because of their interest in the flow of time. It is something that has to eventually be incorporated into their actual main point.

So while I totally agree that A sounds like support for a resolution of the problem presented in D, D still can't be the answer. We can't pick B, C or E either. We are left with A, for which I can at least present some rationale. This is my best stab at it off the top of my head. Hopefully it helps.

LSATMoron

New
Posts: 4
Joined: Sun Feb 24, 2019 10:20 am

Re: PT 86 - Multiverse - Q20

Post by LSATMoron » Sat Oct 12, 2019 10:36 pm

Blueprint LSAT wrote:I definitely see your logic here. P5 does sound a lot like they are just clearing up a potential problem with the theory. I think the best way to get where we need to go is to look at which of the answer choices is the best summary of the point C&C are trying to make.

Here are all the claims made by C&C:
P2: The Big Bang was not a unique event; events like it happen periodically over an incredibly vast time
scale.
P4: The most common initial condition is actually likely to resemble cold, empty space.
P5: Our universe and others are fluctuations in a high entropy multiverse.

If you think of the statement in P5 as just summarizing how they are solving the problem presented in P4 then doesn't sound like a good candidate for the main point.

However, if you think of it as an updated paraphrasing of the overall claim after accounting for that problem and solution it makes a little more sense I think.

We have two seemingly contradictory hypotheses:
The starting conditions should be low-entropy
The Big Bang was not a unique event

Some evidence is added to reconcile these two expectations and the explanation we are left with is that our universe and others are fluctuations in a high entropy multiverse. That is the explanation the author goes on to agree with, which makes A the best candidate among our answer choices for our main point.

I don't love it but it checks.

The problem with D is that even if A was support for it it would then still be support for something else (probably the original claim in P2). C&C's overall claim isn't that high entropy is the likeliest starting condition. That is a complication they became interested in because of their interest in the flow of time. It is something that has to eventually be incorporated into their actual main point.

So while I totally agree that A sounds like support for a resolution of the problem presented in D, D still can't be the answer. We can't pick B, C or E either. We are left with A, for which I can at least present some rationale. This is my best stab at it off the top of my head. Hopefully it helps.
Thank you! This question was confusing the hell out of me and I am grateful for your detailed explanation. Really helped me understand where I was going wrong. Thanks again!

Post Reply

Return to “LSAT Prep and Discussion Forum”